This doesn't seem too likely

Algebra Level 4

What is the largest integer n 1000 n \leq 1000 , such that there exist 2 non-negative integers ( a , b ) (a, b) satisfying

n = a 2 + b 2 a b 1 ? n = \frac{ a^2 + b^2 } { ab - 1 } ?

Hint : ( a , b ) = ( 0 , 0 ) (a,b) = (0,0) gives us 0 2 + 0 2 0 × 0 1 = 0 \frac{ 0^2 + 0^2 } { 0 \times 0 - 1 } = 0 , so the answer is at least 0. 0 .


The answer is 5.

This section requires Javascript.
You are seeing this because something didn't load right. We suggest you, (a) try refreshing the page, (b) enabling javascript if it is disabled on your browser and, finally, (c) loading the non-javascript version of this page . We're sorry about the hassle.

8 solutions

Ariel Gershon
Oct 31, 2014

Lemma 1:

Suppose a certain pair of integers ( a , b ) (a,b) is a solution to the equation n = a 2 + b 2 a b 1 n = \frac{a^2+b^2}{ab-1} . Then I say the pair ( b , b 2 + n a ) (b, \frac{b^2+n}{a}) is also an integer solution.

Proof:

Consider the equation x 2 n b x + ( b 2 + n ) = 0 x^2 - nbx + (b^2+n) = 0 . By assumption, a a is one of the roots. Therefore, by Vieta's formulas, the other root is b 2 + n a \frac{b^2+n}{a} . By substituting the formula for n n , this equals a 2 + a b 3 a ( a b 1 ) \frac{a^2+ab^3}{a(ab-1)} . We already know that the numerator is divisible by a b 1 ab-1 (since n n is an integer), and it's clearly also divisible by a a . Since g c d ( a , a b 1 ) = 1 gcd(a,ab-1) = 1 , then the numerator is also divisible by a ( a b 1 ) a(ab-1) . Hence the new root is an integer. Hence, ( b , b 2 + n a ) (b, \frac{b^2+n}{a}) is also an integer solution. to the equation, \boxed{ }

Lemma 2:

If a > b 3 a > b \ge 3 , then b 2 + n a < b \frac{b^2+n}{a} < b .

Proof:

Since b 3 b \ge 3 , then 2 b b 2 1 < 1 \frac{2b}{b^2-1} < 1 . Therefore, b 3 + b b 2 1 = b + 2 b b 2 1 < b + 1 a \frac{b^3+b}{b^2-1} = b + \frac{2b}{b^2-1} < b +1 \le a b 3 + b < a b 2 a b^3+b < ab^2-a a + b 3 < a b 2 b a+b^3 < ab^2-b a + b 3 a b 1 < b \frac{a+b^3}{ab-1} < b b 2 + n a < b \frac{b^2+n}{a} < b Hence b 2 + n a < b \frac{b^2+n}{a} < b . \boxed{ }

Now, let ( a , b ) (a,b) be the solution to n = a 2 + b 2 a b 1 n = \frac{a^2+b^2}{ab-1} such that the value of b b is minimized (but nonnegative). Then, by Lemmas 1 and 2, we must have 0 b 2 0 \le b \le 2 , otherwise it is possible to get a smaller value of b b .

If b = 0 b = 0 , then n = a 2 0 n = -a^2 \le 0 .

If b = 1 b = 1 , then n = a 2 + 1 a 1 = a + 1 + 2 a 1 n = \frac{a^2+1}{a-1} = a+1 + \frac{2}{a-1} . Then a = 0 , 2 , 3 a = 0,2,3 and therefore n = 1 , 5 , 5 n = -1, 5,5 respectively.

If b = 2 b = 2 , then we have n = a 2 + 4 2 a 1 n = \frac{a^2 + 4}{2a-1} . Thus 4 n = 4 a 2 + 16 2 a 1 = 2 a + 1 + 17 2 a 1 4n = \frac{4a^2+16}{2a-1} = 2a+1+\frac{17}{2a-1} . Hence, 2 a 1 2a-1 is a factor of 17. Therefore a = 1 , 9 a = 1, 9 , and therefore n = 5 n = 5 .

Therefore, the maximum possible value of n n in any of these cases is 5 (and we don't even need the restriction that n 1000 n \le 1000 ). Hence the answer is 5 \boxed{5} .

This is great! Could you add it to the wiki page Vieta Root Jumping ? It was included as the last example, with a solution pending.

Calvin Lin Staff - 6 years, 7 months ago

I like your observations!

Why would the minimum value of b provide the maximum value for n?

Steven Perkins - 6 years, 7 months ago

Log in to reply

That's not what I'm saying. What I proved is that, if the value of b b is at least 3, then it's always possible to find a smaller one (for the same value of n n ). Therefore, it's possible to keep reducing until we get a pair ( a , b ) (a,b) where 0 b 2 0 \le b \le 2 for the same value of n n . Therefore, we only need to check the cases where 0 b 2 0 \le b \le 2 .

Ariel Gershon - 6 years, 7 months ago

It's seems, moreover, that n=5 is the only possible integer solution of this equation, for any a,b. So that terms as "minor then" or "maximum" are irrilevant.

BUT, Ido not know why.....

Luciano Riosa - 6 years, 7 months ago

First, for the equation you assumed let /('a'/) be a root. Then you worked out the other root as b 2 + n / a {b^2 + n}/a an later stated b , b 2 + n / a b,{b^2 + n}/a as the two integer solution.

Can you also elaborate how 'b' greater than 3 implies that it could be reduced to a lower value for same 'n'. I'm new to this concept.

Vishal Yadav - 4 years, 3 months ago

Log in to reply

Oh yes. Suppose you start with a pair of integers ( a , b ) (a,b) which are a solution to the equation. Perhaps I should have mentioned that a a cannot equal b b , because that would imply the following: n = 2 a 2 a 2 1 = 2 + 2 a 2 1 n = \dfrac{2a^2}{a^2-1} = 2 + \dfrac{2}{a^2-1} which would make n n not an integer.

Hence either a > b a >b or b > a b > a , and since order doesn't matter, we can assume that a > b a > b .

Now, in my first lemma, I showed that ( b , b 2 + n a ) \left(b, \dfrac{b^2+n}{a}\right) is also a solution to the same equation. And in my second lemma, I showed that if b 3 b \ge 3 then b 2 + n a \dfrac{b^2+n}{a} is strictly less than b b . So, whenever b b is at least 3, this new solution that we get in the first lemma has a lower second value. So, instead of considering all possible values of b b we just need to consider b = 0 , 1 b = 0, 1 or 2 2 .

Ariel Gershon - 4 years, 3 months ago

Great proof! If I'm not mistaken, we still need to show there is no solution if we let 3 b = a 3\leq b=a . The Vièta-Root-Jumping part only applies to 3 b < a 3\leq b\boxed{<}a and (sadly) cannot be extended to b = a b=a . Afterwards, this case is never considered.

For a (slightly) different approach, hopefully covering all cases, see my solution below.

Carsten Meyer - 2 months ago
Trevor Arashiro
Oct 28, 2014

Begin by moving everything on one side of the equation

a 2 n a b + n + b 2 a^2-nab+n+b^2

Use the quadratic equation to solve for a.

a = n b ± n 2 b 2 4 n 4 b 2 2 a=\dfrac{nb\pm \sqrt{n^2b^2-4n-4b^2}}{2}

Now we need to check integer solutions for the radical.

Note: if the first term of a square root of a second degree polynomial is a perfect square, then the sum of other terms must be a difference of squares.

Thus we can set our radical equal to ( n a b + p ) (nab+p) .

( n a b + p ) 2 = n 2 a 2 b 2 4 n 4 b 2 (nab+p)^2=n^2a^2b^2-4n-4b^2

2 n a b p + p 2 = 4 n 4 b 2nabp+p^2=-4n-4b

n = ( p 2 + 4 b ) 2 a b p + 4 n=\dfrac{-(p^2+4b)}{2abp+4}

We will use induction to find cases from here

for n to be an integer, p must be even.

since a,b are positive, to maximize n; p must be negative

to maximize (a,b), |p| must be minimized.

to maximize n; b must be maximized

to maximize b; a must be minimized

Thus plugging in p=-2, a=1 we get

n = 4 ( b 2 + 1 ) 4 ( b 1 ) n=\dfrac{-4(b^2+1)}{-4(b-1)}

n = b 2 + 1 b 1 n=\dfrac{b^2+1}{b-1}

This obviously doesn't have any solutions for b>3 since b 2 + 1 2 ( m o d b 1 ) b^2+1\equiv 2 \pmod{b-1}

EDIT (Thanks to Sir Calvin):

The proof for this comes from the partial fraction decomposition of the expression above to n = b + 1 + 2 b 1 n=b+1+\dfrac{2}{b-1} , for which the last term is a non-integer for b>3.

Thus plugging in a and b into our original equation, n = 5 \boxed{n=5}

At the last step, do the typical partial fractions expression to get

n = b + 1 + 2 b 1 n = b + 1 + \frac{2}{ b-1}

And it becomes clear that b 1 2 b -1 \mid 2 , and the result follows.

Calvin Lin Staff - 6 years, 7 months ago

Shouldn't it be a = n b ± n 2 b 2 4 n 4 b 2 2 a = \dfrac {nb \pm \sqrt {n^2b^2 - 4n -4b^2 } } {2} ?

Chew-Seong Cheong - 6 years, 7 months ago

Log in to reply

Thank you, it is now.

Trevor Arashiro - 6 years, 7 months ago

Log in to reply

I think you still need to delete away two more a 2 a^2 's and four a a 's.

Chew-Seong Cheong - 6 years, 7 months ago

How about (a, b) of {(1538, 7369), (4729, 22658), (7369, 35307), (3883449, 18606722) and etc?} The only thing is n is not available with value other than 5. n = (b^2 + 1)/ (b - 1) seem to explain logically, I feel you haven't explained properly by every step you implied. (1, 2), (1, 3), (2, 9) and (3, 14) for example can be (b, a) instead of (a, b), while they can be true to your equation occasionally only when the right b is meant.

Lu Chee Ket - 6 years, 5 months ago

In the line with the (Na+p)^2, on the right side, why is there a a^2 term? Can u also clarify the statement after the word "note"? Thanks!!!

Willia Chang - 5 years ago
Calvin Lin Staff
Oct 27, 2014

[This is not a complete solution]

The case when a b = 0 ab = 0 is easily dealt with, and we have n 0 n \leq 0 .

If a a and b b are positive integers such that a 2 + b 2 a b 1 = k \frac{ a^2 + b^2 } { ab-1 } = k is an integer, then k = 5 k = 5 .

If you can prove this, I would appreciate if you can add your solution to the Vieta Root Jumping Wiki page.

Sir @Calvin Lin please tell me an appropriate proof for this without calculus and all ,as I'm just 15(not 19)

Mehul Chaturvedi - 6 years, 7 months ago

Log in to reply

Note that not all mathematical statements can be proofed using "solutions appropriate for 15 year olds". This is especially true in Number Theory, with Fermat's Last Theorem being a classic example.

Trevor's solution doesn't use calculus, and only uses elementary Number Theory approaches.

As mentioned, there is a solution which uses Vieta Root Jumping. Read the wiki article to understand more.

Calvin Lin Staff - 6 years, 7 months ago

EYYYYYYYYYY me too, I'm 15 but my profile says 19 😂.

Trevor Arashiro - 6 years, 7 months ago

Log in to reply

@Trevor Arashiro @Mehul Chaturvedi I have updated your ages to 15.

Calvin Lin Staff - 6 years, 7 months ago

Log in to reply

@Calvin Lin Thank you Sir

Mehul Chaturvedi - 6 years, 7 months ago

Hey brother please suggest me some book for NUMBER THEORY (actually I'm quite weak at it)

Mehul Chaturvedi - 6 years, 7 months ago
Carsten Meyer
Feb 24, 2019

Notice the symmetry: If ( a , b ) (a,\:b) is a solution to any given n n , so is ( b , a ) (b,\:a) . We only need to consider 0 a b 0\leq a\leq b

  • Check some special cases with partial fraction decomposition (PFD). We must ignore a = b = 1 \red{a=b=1} , or we would divide by zero: 0 = a b : n = b 2 ! Z b 0 , n 0 1 = a < b : n = b 2 + 1 b 1 = b + 1 + 2 b 1 ! Z b { 2 ; 3 } , n = 5 2 a = b : n = 2 a 2 a 2 1 = 2 + 2 a 2 1 ∉ Z 2 a = b 1 : n = a 2 + ( a + 1 ) 2 a ( a + 1 ) 1 = 2 + 3 a ( a + 1 ) 1 ∉ Z \begin{aligned} 0 &= a\leq b: &&& n &= -b^2\overset{!}{\in}\mathbb{Z} &&& \Rightarrow &&&& b &\geq 0, & n&\leq 0 \\[1em] 1 &= a\red{<} b: &&& n &= \frac{b^2+1}{b-1}=b+1+\frac{2}{b-1}\overset{!}{\in}\mathbb{Z} &&&\Rightarrow &&&& b&\in\{2;\:3\}, &n&=5\\[1em] 2 &\leq a=b: &&& n &= \frac{2a^2}{a^2-1}=2 + \frac{2}{a^2-1}\not\in\mathbb{Z}\\[1em] 2 &\leq a=b-1: &&& n &= \frac{a^2+(a+1)^2}{a(a+1)-1}=2 + \frac{3}{a(a+1)-1}\not\in\mathbb{Z}\\[1em] \end{aligned} In the last two cases, the denominator of the remainder is greater than the numerator and cannot generate an integer. The only remaining case we still need to check is 2 a b 2 2\leq a\leq b-2

  • Claim: Let 2 a b 2 2\leq a\leq b-2 . If ( a , b ) (a,\:b) is a solution to any given n n , so is ( a , b ) : = ( n a b , a ) (a',\:b'):=(na-b,\:a) with 0 < a < a 0<a'<a . Proof: If ( a , b ) (a,\:b) is such a solution, then n > 0 n>0 . We can write b b as a root of a polynomial P ( x ) P(x) : n = a 2 + b 2 a b 1 Z 0 = a 2 + b 2 n ( a b 1 ) = b 2 n a b + a 2 + n = : P ( b ) \begin{aligned} n&=\frac{a^2+b^2}{ab-1}\in\mathbb{Z} &&&\Rightarrow &&&& 0&=a^2+b^2-n(ab-1)=b^2-nab + a^2+n=:P(b) \end{aligned} With Vieta Root Jumping , we can find a second root b b^* of P ( x ) P(x) : b + b = n a a = b = n a b Z , a = b = a 2 + n b > 0 + 0 = 0 \begin{aligned}b^*+b & = na &\Rightarrow &&a'&=b^*= na-b\in\mathbb{Z}, &&&&& a' &=b^*= \frac{a^2+n}{b}>0+0=0 \end{aligned} The other inequality a < a a'<a is a bit harder to prove. Insert the definition of n n into a a' and get a = a 2 + b 2 a b 1 a b = a 3 + b a b 1 ( ) a 3 + ( a + 2 ) a ( a + 2 ) 1 = PFD a 2 + 6 a a ( a + 2 ) 1 ( ) a 2 + 12 7 < a \begin{aligned} a'&=\frac{a^2+b^2}{ab-1}\cdot a-b=\frac{a^3+b}{ab-1}\underset{(*)}{\leq}\frac{a^3+(a+2)}{a(a+2)-1}\underset{\text{PFD}}{=}a-2+\frac{6a}{a(a+2)-1}\underset{(**)}{\leq} a-2+\frac{12}{7}<a \end{aligned} In ( ) (*) , we use that the expression is decreasing in b a + 2 b\geq a+2 and a 2 a\geq 2 . In ( ) (**) the remainder is decreasing in a 2 a\geq 2\quad\blacksquare

  • We still consider 2 a b 2 2\leq a\leq b-2 . For any solution ( a , b ) (a,\:b) to a given n n , we can repeat the last step to generate a sequence of solutions with decreasing a > 0 a'>0 . After a finite number of steps, we have a = 1 a'=1 , the second of our special cases! Every non-negative solution can be reduced to one of the special cases, so the maximum value is n = 5 n=\boxed{5}

Rem.: We even showed that n { 0 ; 5 } n\in\{0;\:5\} are the only non-negative solutions to the problem! Both fundamental solutions with a = 1 a=1 and b { 2 ; 3 } b\in\{2;\:3\} generate a countable sequence of solutions with n = 5 n=5 . That was fun :)

Every number can be of the form 3k,3k+1,3k+2.

So using this we can find maximum value of n to be 5.

Sorry this is a wrong way to get answer.

The reason why I am not deleting this solution is there may be some one who thought the same like me so this may be helpful.😁

Can you elaborate further? How does this lead you to conclude that the largest integer value of n is 5?

Note that there is no maximum (non-integer) value of n. For example, with b = 1 b = 1 , we have n = a 2 + 1 a 1 = a + 1 + 2 a 1 n = \frac{ a^2 + 1 } { a - 1 } = a + 1 + \frac{ 2 } { a-1} , which has no maximum.

Calvin Lin Staff - 4 years ago

As n is also integer we can get that directly , plz substitute a,b as 3k,3k+1,3k+2.(a,b differ in k) and you will notice that the latest integral value of n to be equal to 5.

Sudhamsh Kukkadapu - 4 years ago

Log in to reply

Can you be explicit about what you're substituting for a and then for b? Why must "a,b differ in k".

For example, note that a = 9 , b = 2 a = 9 , b = 2 leads to an integer n = 5 n = 5 . What does this correspond to in "plz substitute a,b as 3k,3k+1,3k+2.(a,b differ in k)"?

Calvin Lin Staff - 4 years ago

every square number will be of the form 3k or 3k+1, so sum of two squares can be of the form 3m,3m+1,3m+2

Case 1

Both are of the form 3k+1 ,

So ab-1 is of the form 3n

So we get (3k'+2)/3n to be equal a 2 + b 2 a^2+b^2 /ab-1 that can never be an integer .

Case 2

One of the form 3k , 3k+1

So ab-1 is again of the form 3n+2,

And a 2 + b 2 a^2+b^2 is of the form 3k'+1 which again does not give an integer

Since 3k'+1/3n+2 ≠ integer .

Case 3

Both of the form 3k,3k

We get ab-1 of the form 3n+2

And a²+b² is of the form 3k".

Like this you continue the process to get an integer value.

Sudhamsh Kukkadapu - 4 years ago

Log in to reply

When you say "both are of the form 3k+1", are you setting a = 3 k + 1 a = 3k + 1 and b = 3 k + 1 b = 3k + 1 ? If so, does that make sense to you that we're allowed to do so?

I'm not sure how you concluded that "ab-1 is of the form 3n". I agree that " a 2 + b 2 a^2+b^2 is of the form 3k+2 (for some k). However, why does that imply that ab-1 must be a multiple of 3? In fact, it tells us that ab-1 cannot be a multiple of 3.


Can you use the example of a = 9 , b = 2 a = 9, b = 2 to walk me through your calculations? E.g. set a = 3 × 3 + 0 , b = 3 × 0 + 2 a = 3 \times 3 + 0, b = 3 \times 0 + 2 , then what happens?

Calvin Lin Staff - 4 years ago

Ah ic. Thanks for explaining the step that you want to show " a^2+b^2 / ab-1 can never be an integer". Previously, I was working under the condition in the problem that it had to be an integer, which is why we have ab-1 divides 3K+2.

This is why it is important to be clear about what you are doing, and what you want to show. Despite technological advances, not everyone is a mind reader.


  1. Case 2: It is not a true statement that "3k'+1 / 3n+2 is never an integer". For example, take k=5, n = 2 (your notation of n, not the problems notation). We have 16 / 8 = 2.
    In particular, this is what happened with a = 9 , b = 2 a = 9, b = 2 , where a 2 = 3 k , b 2 = 3 k + 1 a^2 = 3k, b^2 = 3k+1 . and we get an integer. This is why I ask you to "use this example to walk me through your calculation"

  2. Case 3: I'm not sure what you mean by "Like this you continue the process to get an integer value".

  3. Finally, how do you conclude that the maximum value of n is 5? I do not see 5 appear anywhere, nor do I see the justification that it is the largest.

Calvin Lin Staff - 4 years ago

Log in to reply

Oh, I understood what mistake I have done

Thank you so much for discussion sir , @Calvin Lin

Sudhamsh Kukkadapu - 4 years ago

@Calvin Lin , Consider a=3k+1, b=3l+1,

So ab = (3k+1)(3l+1) = 9kl + 3l + 3k +1 ,

So clearly we see that ab-1 is multiple of 3.

Sudhamsh Kukkadapu - 4 years ago
Lu Chee Ket
Jan 4, 2015

3 tries available despite incorrect answer. Searched up to 3 millions for n up to 1000 and 30 millions for n up to 100 via another modified parameters. Found that (n^2 - 4) b^2 and 4 n are to split after a formation for n = 5; such split makes possibility of formation of perfect square getting further; though, I consider my answer as a guess. There are many pairs of (a, b) along for n = 5 found by initial routine before the modification as follows. By rule of beauty, the most possible situation for search beyond 3 millions for n up to 1000 would not produce other wanted finding.

PROGRAM Largest_Integer;

USES CRT;

VAR

a, b, d, n, p: EXTENDED;

BEGIN

CLRSCR;

 n:=1000.0;

 REPEAT

     b:=2.0;

     REPEAT

           p:=b*n;

           d:=p*p-4.0*(b*b+n);

           IF d>=0 THEN

           BEGIN

                a:=0.5*(p+SQRT(d));

                IF (FRAC(a)=0.0) AND (n<>5.0) THEN

                BEGIN

                     WRITELN('Finished.');

                     WRITELN(n:1:0, '   ', a:1:6, '   ', b:1:0);

                     WRITE((a*a+b*b)/(a*b-1.0):1:18);

                     READLN;

                     EXIT

                END

           END;

           IF b=2999999.0 THEN

              WRITELN(n:1:0, '   ', a:1:6, '   ', b:1:0);

           b:=b+1.0

     UNTIL b=3000000.0;

     n:=n-1.0

 UNTIL n=2

END.

Note that this does not constitute a proof by exhaustion.

Calvin Lin Staff - 6 years, 5 months ago

Log in to reply

Not a proof. However, many questions involving attempt to find for valid possibility to infinity can be indicated by certain extremes to the limit via computing search. Ancient tradition is conceptually good while modern approach can be fast and general when intuitive mind has been developed.

Lu Chee Ket - 6 years, 5 months ago
Anna Anant
Nov 13, 2014

Consider the quadratic equation, x^2 - nbx + b^2 + n = 0. We know that one solution is a, and the other is c:= (b^2 + n)/a. From the equation on the problem, we let a = 1. So that n = (1+b^2)/(b-1) = (2+ b^2 - 1)/(b-1). Since b is a positive integer, then b-1 is also a positive integer. Since b^2 -1 is always divisible by b-1, and n is integer, then 2 must be divisible by b-1. From this we have b= 2 or b= 3. Since a=1 and b=2 then n=5. And since a=1 and b=3, we also have n=5. So the maximum of n is 5.

Mehul Chaturvedi
Oct 28, 2014

W e m a y u s e i n d u c t i o n a s f o l l w s 1 ) l e t a = 1 b = 3 T h e r e f o r e n = 5 2 ) l e t a = 2 b = 9 t h e r e f o r e n = 5 . . . . m e a n s b w e v e r y 10 n o s l i k e 1 10 , 11 20.....991 1000 i t s w i l l g i v e o n e v a l u e a s " 5 " " ( I k n o w i t s j u s t t h e t r i o c k w e c a n u s e i n M C Q s ) n e v e r i n c l u d e t h i s i n p r o o f s We\quad may\quad use\quad induction\quad as\quad follws\\ 1)let\quad a=1\quad b=3\quad Therefore\quad n=5\\ 2)let\quad a=2\quad b=9\quad therefore\quad n=5\\ .\\ .\\ .\\ .\\ means\quad bw\quad every\quad 10\quad nos\\ like1-10,11-20.....991-1000\\ its\quad will\quad give\quad one\quad value\quad as\quad "'5""\\ \\ (I\quad know\quad its\quad just\quad the\quad triock\quad we\quad can\\ use\quad in\quad MCQs)\\ never\quad include\quad this\quad in\quad proofs

I do not see how you used induction. Can you explain your steps further?

The issue is that you do not know why no higher number other than 5 cannot be achieved.

Calvin Lin Staff - 6 years, 7 months ago

Log in to reply

Yes its the highest integral value actually i saved some steps by not substituting the values which would give non integral solutions

Mehul Chaturvedi - 6 years, 7 months ago

Log in to reply

i just put the values from 1-20 and surprisingly got 5 only both the times b/w 1-10 and 11-20 (and by the way these are some basic observations)

Mehul Chaturvedi - 6 years, 7 months ago

Log in to reply

@Mehul Chaturvedi Right. So the concern is that there could be other bigger values, like a = 123456789 a = 123456789 and b = 987654321 b = 987654321 which give a different value of n n .

Calvin Lin Staff - 6 years, 7 months ago

Log in to reply

@Calvin Lin By the way in simple one step induction we do not put values from 0-10000000000...... and actually for a=123456789... b=987654321.. it would give us a non integral value

WE OBSERVE THAT b/w every ten nos starting from 1-10 we would always get max. integral value as 5

Mehul Chaturvedi - 6 years, 7 months ago

0 pending reports

×

Problem Loading...

Note Loading...

Set Loading...